Permutations with arithmetic constraints

arxiv(2022)

引用 1|浏览2
暂无评分
摘要
Let $S_{\rm lcm}(n)$ denote the set of permutations $\pi$ of $[n]=\{1,2,\dots,n\}$ such that ${\rm lcm}[j,\pi(j)]\le n$ for each $j\in[n]$. Further, let $S_{\rm div}(n)$ denote the number of permutations $\pi$ of $[n]$ such that $j\mid\pi(j)$ or $\pi(j)\mid j$ for each $j\in[n]$. Clearly $S_{\rm div}(n)\subset S_{\rm lcm}(n)$. We get upper and lower bounds for the counts of these sets, showing they grow geometrically. We also prove a conjecture from a recent paper on the number of "anti-coprime" permutations of $[n]$, meaning that each $\gcd(j,\pi(j))>1$ except when $j=1$.
更多
查看译文
关键词
constraints
AI 理解论文
溯源树
样例
生成溯源树,研究论文发展脉络
Chat Paper
正在生成论文摘要